Answer (C) is correct . The finished units needed for sales (8,000), plus the units desired for ending inventory (12,000 units to be sold in the third quarter ¡Á 20% = 2,400), minus the units in beginning inventory (8,000 units to be sold in the second quarter ¡Á 20% = 1,600), equals budgeted production for the second quarter of 8,800 units.Answer (A) is incorrect because Subtracting the beginning inventory twice results in 7,200 units. Answer (B) is incorrect because Assuming no change in inventory results in 8,000 units. Answer (D) is incorrect because Including the beginning inventory for the first quarter, not the second quarter, in the calculation results in 8,400 units.
|